logo-type-white

AP® English Language

3 ap® english language rhetorical essay strategies.

  • The Albert Team
  • Last Updated On: March 1, 2022

3_ap_english_language rhetorical essay strategies

The AP® English Language rhetorical essay can be nightmare inducing for some AP® students, but there is no need for fear. In this exam review we will lay out helpful strategies to get you through the rhetorical essays in no time.

Rhetorical Strategy #1: Dissecting the Prompt

The first rhetorical essay strategy is to dissect the prompt. Understanding what the rhetorical essay wants from you is essential. It is important for you to read the prompt carefully for every essay, but critical reading is even more essential to the rhetorical essay. Your rhetorical prompt that you will be given for the AP® English Language exam will contain two elements. The first element is the concrete task that the prompt is asking of you, which is always to analyze the passage that follows. The second part of the prompt is a more abstract task that is not directly asked for in the prompt, but it is implied. By completely understanding both parts of the prompt, you will be able to give a complete essay that will get you to a higher score.

One example of a prompt from an AP® English Language rhetorical essay is this one from the 2008 exam . The prompt reads:

“In the following passage from The Great Influenza, an account of the 1918 flu epidemic, author John M. Barry writes about scientists and their research. Read the passage carefully. Then, in a well-written essay, analyze how Barry uses rhetorical strategies.”

Here you can see the concrete task that the examiners are asking. They want you to analyze the passage for rhetorical strategies; however, you must figure out what you are analyzing the passage for. That is the more abstract concept that you need to dissect the prompt to find. In the case of Barry’s passage you will need to analyze how he uses rhetorical strategies in order to portray scientific research. We know this, because if you look at the prompt, it specifically states what Barry did in his work, which was to write about science and research. That is your abstract task.

Once you have found your concrete task and your abstract task, a great strategy is to write it down to keep you focused throughout your essay . Using the example above this would look like the following:

Analyze how Barry uses rhetorical strategies in order to portray scientific research.

That sentence is what you must follow when writing your essay, and if you successfully keep to this task, then you will move closer to that high score.

Rhetorical Essay Strategy #2: Stick to the Format

This next rhetorical essay strategy is the key to great organization and structure that will put your test anxiety to bed. There is a simple paragraph structure for the body paragraphs of the AP® English Language rhetorical essay that will allow you to think, write, and score higher, faster. You need to begin each body paragraph with an assertion or claim. That is the point that you are trying to make clear to your audience what you will be proving. A great example of this is from the 2006 AP® English Language rhetorical essay. Below is student 2B’s opening sentence for her first body paragraph.

“The diction of the passage fully relays Hazlitt’s position about money ( student 2B ).”

You can see how the student directly asserts what he or she will be proving in this statement. The next step in constructing your body paragraph is to give one to two pieces of textual evidence. Be sure to state why these quotations relate back to your claim, otherwise they will be deemed irrelevant by the examiners. An example of this is the next sentence in student 2B’s body paragraph about diction. Here, the student brings in elements from the text to support his or her claim about Hazlitt using diction.

“’Rejected’, ‘contempt’, ‘disparaged’, ‘scrutinized’, ‘irksome’, ‘deprived’, ‘assailed’, ‘chagrin’; the endless repetition of such discouragement shows just how emphatically Hazlitt money is requisite for happy life (student 2B).”

The final part of this strategy for conquering the body paragraphs of your rhetorical essays is to end those body paragraphs with a thorough analysis. This is the aspect of the exam where you can put your way of looking at the text into your essay.

An example of this is at the end of student 2B’s body paragraph where he or she states, “The irony of the last sentences is negative, conveying the utter hopelessness of one without money. Though one may have none in life, pitiless men will continue to mock one’s circumstances even after death! (student 2B)”

This analysis of the text adds to the textual examples above and continues to bring in new logic from the student.

When this format of a body paragraph is followed, then it is extremely effective. The essay becomes clear, assertive, and easy to follow for the examiners. Follow this rhetorical essay strategy and you are even closer to getting that 5 on the exam.

Rhetorical Essay Strategy #3: LORA

As you are looking at your AP® English Language rhetorical essay prompt and passage it is important to remember the mnemonic device, LORA. LORA stands for Language, Organization, and Rhetorical Appeals. These elements will help you form your argument.

When you read through your passage you want to think about how the author is utilizing language. Is he or she using figurative language effectively? Is there imagery within the passage? Does the diction of the passage make it more rhetorically persuasive? You should not use all of these, but picking one and analyzing it clearly in one paragraph will keep you focused on how the author uses rhetoric, which is the main task of this essay.

An example of this was in the 2006 AP® English Language rhetorical essay. Student 2A begins his or her first body paragraph with, “One of Hazlitt’s most effective methods of promoting the importance of money is his strong diction (student 2A).” This student begins his or her essay with focusing on diction as how the language is used. He or she then goes on to explain why diction betters Hazlitt’s argument, which is exactly what you must do for your own rhetorical essay.

The organization of the author is the next part of your answer to the prompt. You want to look at how the author organized his or her ideas within the passage to support his or her own argument. By pointing out the organization, or structure, of the work and how it adds to the overall persuasiveness, you will bring two of the three most important elements of rhetoric together in your essay.

After organization you need to look at the rhetoric appeals. You may know them by the names logos, pathos, and ethos. It is suggested that you cover as many of these as possible; however, if time does not permit or if the passage uses one more than the other, then you should focus on one appeal.

One example of using pathos in an essay is from student 2A from the 2006 prompt. “Hazlitt plays on the audience’s heartstrings for more than enough time to convince them of the importance of having money (student 2A).” While it would have been better for the student to directly say that this is pathos, he or she does thoroughly explain the appeal to the passions, or pathos.

Key Takeaways

When taking the AP® English Language rhetoric essay you just need to remember these three rhetorical essay strategies: dissect the prompt, follow the format, and always include LORA. If you can follow them, then you are already on your way to a 5 on the AP® English Language exam .

Let’s put everything into practice. Try this AP® English Language practice question:

Rhetorical Considerations AP® English Language Practice Question

Looking for more AP® English Language practice?

Check out our other articles on AP® English Language .

You can also find thousands of practice questions on Albert.io. Albert.io lets you customize your learning experience to target practice where you need the most help. We’ll give you challenging practice questions to help you achieve mastery of AP® English Language.

Start practicing here .

Are you a teacher or administrator interested in boosting AP® English Language student outcomes?

Learn more about our school licenses here .

Interested in a school license?​

Popular posts.

AP® Physics I score calculator

AP® Score Calculators

Simulate how different MCQ and FRQ scores translate into AP® scores

ap lang rhetorical essay prompts

AP® Review Guides

The ultimate review guides for AP® subjects to help you plan and structure your prep.

ap lang rhetorical essay prompts

Core Subject Review Guides

Review the most important topics in Physics and Algebra 1 .

ap lang rhetorical essay prompts

SAT® Score Calculator

See how scores on each section impacts your overall SAT® score

ap lang rhetorical essay prompts

ACT® Score Calculator

See how scores on each section impacts your overall ACT® score

ap lang rhetorical essay prompts

Grammar Review Hub

Comprehensive review of grammar skills

ap lang rhetorical essay prompts

AP® Posters

Download updated posters summarizing the main topics and structure for each AP® exam.

Interested in a school license?

ap lang rhetorical essay prompts

Bring Albert to your school and empower all teachers with the world's best question bank for: ➜ SAT® & ACT® ➜ AP® ➜ ELA, Math, Science, & Social Studies aligned to state standards ➜ State assessments Options for teachers, schools, and districts.

Fiveable

Find what you need to study

Rhetorical Analysis Essay How-To

7 min read • november 18, 2021

Kathryn Howard

Kathryn Howard

Brandon Wu

Mixed AP Review

Endless stimulus-based MCQs for all units

ap lang rhetorical essay prompts

In these three things—production, with the necessity of exchanging products, shipping, whereby the exchange is carried on, and colonies, which facilitate and enlarge the operations of shipping and tend to protect it by multiplying points of safety—is to be found the key to much of the history, as well as of the policy, of nations bordering upon the sea. The policy has varied both with the spirit of the age and with the character and clear-sightedness of the rulers; but the history of the seaboard nations has been less determined by the shrewdness and foresight of governments than by conditions of position, extent, configuration, number and character of their people,—by what are called, in a word, natural conditions.

ap lang rhetorical essay prompts

What is the Rhetorical Analysis Essay?

According to the college board:.

 “The rhetorical analysis free-response essay question presents students with a passage of nonfiction prose of approximately 600 to 800 words. Students are asked to write an essay that analyzes the writer’s rhetorical choices . This question assesses students’ ability to do the following:

Respond to the prompt with a thesis that analyzes the writer’s rhetorical choices .

Select and use evidence to support your line of reasoning .

Explain how the evidence supports your line of reasoning .

Demonstrate an understanding of the rhetorical situation .

Use appropriate grammar and punctuation in communicating your argument."

Essentially, you are being asked to analyze someone’s writing and what strategies they used to help them achieve their purpose.

Rhetorical Analysis Rubric for Scoring

The rhetorical analysis frq is out of 6 points.

https://i.ibb.co/5symGXr/thesis-statement-woody.jpg

To get this point you need to clearly write a defensible thesis about the rhetorical choices the author makes. Do not take a stance on the argument the author is making if he/she is making one. You are only talking about rhetorical strategies.

🎥 Watch: AP Language - How to Find Rhetorical Strategies

Evidence and Commentary (4 points)

To get the four points you need to not only present evidence but explain why it supports your thesis and how it contributes to the author’s message.

Sophistication (1 point)

To get to this point you have to demonstrate a complex understanding of both what that purpose was, and how the rhetorical analysis devices aided the author’s purpose.

There are a few ways that you can earn the sophistication point :

Explaining the significance or relevance of the writer’s rhetorical choices (given the rhetorical situation ).

Explaining a purpose or function of the passage’s complexities or tensions.

Employing a style that is consistently vivid and persuasive.

You have 40 minutes to complete the rhetorical analysis essay for AP Lang:

12 minutes: Read the text and plan out your essay. (TOBI)

6 minutes: Write your introduction paragraph.

18 minutes: Write 2-3 body paragraphs.

2 minutes: Write a quick conclusion.

2 minutes: Proofread and revise your essay.

🎥 Watch: AP Language - Rhetorical Analysis Organization and Timing

https://i.ibb.co/mCr5BDx/timed.jpg

How to Maximize Your Time

Outline your rhetorical analysis essay before writing! A great tool for this is a TOBI:

TOBI stands for thesis , outline , and big idea.

TOBI Outline

BI -Big Idea

Here is an example of how to use TOBI given a rhetorical analysis prompt:

https://i.ibb.co/hBMWsnj/Screen-Shot-2020-03-08-at-12-44-23-PM.png

From CollegeBoard AP Lang 2017 Exam, FRQ Question 2

T: Luce uses many rhetorical strategies including pathos , antithesis , and a humorous tone to soften up her audience before introducing her true reasons for being there. 

- Pathos Appeal

(“There is no audience more forgiving”)

- Antithesis

(“I am happy, I am less happy”)

- Humorous tone

(“consequently, no audience is more forgiving, I hope”)

BI: Today, just like for Luce, it is very difficult to give criticism to your peers.

Note: It is a good idea to make the TOBI about the size of your hand to make sure you don’t spend too much of your precious essay writing time on it.

What if I can't find any rhetorical devices that I recognize?

You can always go back and rely on tone as every piece of literature has one, even if it is just informative. If you know what they are doing, but not the name of the term, you can still just describe it and get the points. Additionally, make sure that you are familiar with all the rhetorical devices that are a part of AP Lang!

🎥 Watch: AP Language - Reading with an Analytical Mind

If it’s not an argumentative essay, what do you put in your thesis?

You state the most important writing choices the author made in order to impact the audience of the work.

Other Tips and Tricks

The big idea should show how this prompt applies to today. This will help you write your conclusion. In most language arts classes they teach you to simply restate your points, but not in AP Lang!

The first thing you are going to want to do is carefully read through and highlight any strategies you see. 

Even if TOBI doesn’t work for you, it is a good idea to outline the essay. Even though it takes time, it will end up saving you time in the end because it gives you direction.

One of the most useful tools for the introduction is something called Soapstones. In this intro you are introducing the S peaker, O ccasion, A udience, P urpose, S ubject, T one, and S tyle. (Keep in mind: You do not need to include EVERY ONE). But, most successful essays include a few of them.

DON’T SKIM! It will only hurt you in the long run, even if you think it might be saving you time.

If you need to, review strategy names, but if you don’t remember, do your best to describe what is going on and how the author is using it.

https://i.ibb.co/fQfQLNT/diction.jpg

Rhetorical Analysis Example Essay Prompt

The speech below was given at the site of the battle of Gettysburg by president Abraham Lincoln. Lincoln went on to describe his desire to save the union. Read the passage carefully and then in a well-developed essay, analyze the writing choices Lincoln makes to share his message with others. Support your analysis of his rhetoric with specific references from the text.

“Four score and seven years ago our fathers brought forth on this continent a new nation, conceived in Liberty and dedicated to the proposition that all men are created equal. Now we are engaged in a great civil war, testing whether that nation or any nation so conceived and so dedicated, can long endure. We are met on a great battle-field of that war. We have come to dedicate a portion of that field, as a final resting place for those who here gave their lives that that nation might live. It is altogether fitting and proper that we should do this. But, in a larger sense, we can not dedicate—we can not consecrate—we can not hallow—this ground. The brave men, living and dead, who struggled here, have consecrated it, far above our poor power to add or detract. The world will little note, nor long remember what we say here, but it can never forget what they did here. It is for us the living, rather, to be dedicated here to the unfinished work which they who fought here have thus far so nobly advanced. It is rather for us to be here dedicated to the great task remaining before us—that from these honored dead we take increased devotion to that cause for which they gave the last full measure of devotion—that we here highly resolve that these dead shall not have died in vain—that this nation, under God, shall have a new birth of freedom—and that government of the people, by the people, for the people, shall not perish from the earth.”

Key Terms to Review ( 20 )

Complex Understanding

Defendable Thesis

FRQ (Free-Response Question)

Grammar and Punctuation

Humorous Tone

Line of Reasoning

Passage's Complexities or Tensions

Pathos Appeal

Rhetorical Analysis Essay

Rhetorical Analysis Rubric

Rhetorical Choices

Rhetorical Situation

Significance or Relevance

Sophistication Point

TOBI (Thesis, Outline, Big Idea)

Vivid and Persuasive Style

Writer's Rhetorical Choices

Fiveable

Stay Connected

© 2024 Fiveable Inc. All rights reserved.

AP® and SAT® are trademarks registered by the College Board, which is not affiliated with, and does not endorse this website.

AP English Language and Composition: Sample Rhetorical Analysis and Synthesis Questions

August 21, 2021.

The Rhetorical Analysis and Synthesis Essays are two of the three essays you’ll need to write as part of the AP English Language and Composition Exam . Read on for a sample of each, as well as tips for how to answer them. 

AP English Language and Composition: Sample Rhetorical Analysis Question

Read the following passage published back in 1967 by The New York Times. Then write an essay in which you analyze how the structure of the passage and the use of language help convey the writer’s views.

Sample Question Instructions:

  • Respond to the prompt with a thesis that may establish a line of reasoning.
  • Select and use evidence to develop and support the line of reasoning.
  • Explain the relationship between the evidence and the thesis.
  • Demonstrate an understanding of the rhetorical situation.
  • Use appropriate grammar and punctuation in communicating the argument.

Americans and Western Europeans, in their sensitivity to lingering problems around them, tend to make science and progress their scapegoats. There is a belief that progress has precipitated widespread unhappiness, anxieties, and other social and emotional problems. Science is viewed as a cold mechanical discipline having nothing to do with human warmth and the human spirit. 

But to many of us from the nonscientific East, science does not have such repugnant associations. We are not afraid of it, nor are we disappointed by it. We know all too painfully that our social and emotional problems festered long before the age of technology. To us, science is warm and reassuring. It promises hope. It is helping us at long last gain some control over our persecutory environments, alleviating age-old problems—not only physical but also, and especially, problems of the spirit.

Shiraz, for example, a city in southern Iran, has long been renowned for its rose gardens and nightingales; its poets, Sadi and Hafiz; and its mystical, ascetic philosophy, Sufism. Much poetry has been written in glorification of the spiritual attributes of this oasis city. And to be sure, Shiraz is a green, picturesque town, with a quaint bazaar and refreshing gardens. But in this “romantic” city thousands of emotionally disturbed and mentally retarded men, women, and children were, until recently, kept in chains in stifling prison cells and lunatic asylums. 

Every now and again, some were dragged, screaming and pleading, to a courtyard and flogged for not behaving “normally.” But for the most part, they were made to sit against damp walls, their hands and feet locked in chains, and thus immobilized, without even a modicum of affection from their helpless families and friends, they sat for weeks and months and years—often all their lives. Pictures of these wretched men, women, and children can still be seen in this “city of poetry,” this “city with a spiritual way of life.” 

It was only recently that a wealthy young Shirazi who, against the admonitions of his family, had studied psychology at the University of Tehran and foreign universities, returned to Shiraz and after considerable struggle with city officials succeeded in opening a psychiatric clinic, the first in those regions. After still more struggle, he arranged to have the emotionally disturbed and the mentally retarded transferred from prison to their homes, to hospitals, and to his clinic, where he and his staff now attend them. 

They are fortunate. All over Asia and other backward areas, emotionally disturbed men and women are still incarcerated in these medieval dungeons called lunatic asylums. The cruel rejection and punishment are intended to teach them a lesson or help exorcise evil spirits. 

The West, still bogged down in its ridiculous romanticism, would like to believe that emotional disturbances, dope addiction, delinquency are all modern problems brought on by technological progress, and that backward societies are too spiritual and beautiful to need the ministrations of science. But while the West can perhaps afford to think this way, the people of backward lands cannot. . . . 

. . .The obstacles are awesome, the inertia too entrenched, the people’s suffering too anguished, their impatience too eruptive. Moreover, the total cultural reorganizations such as Asia and Africa are undergoing inevitably engender their own temporary dislocations and confusions. But their goals, the direction, remain constant. We are on the move, however awkwardly at first, to a saner, better world.

How to Answer the AP English Language and Composition Rhetorical Analysis Question

Go back to the original question, which asks you to analyze two features of the passage: (1) its structure, or organization, and (2) its language. The first aspect is fairly specific. As you read the passage, you need to observe what the author discusses first, second, third, and so on. Your essay should explain not only the order of ideas but the reasons the author may have chosen that order. 

The second part of the question is more general. It invites you to analyze the use of language, which may include the author’s choice of words (diction), syntax (word order), figures of speech, use of evidence (such as statistics or logical reasoning), sentence structure, rhythm, sound, tone, or just about any other characteristics of style and rhetoric you choose. 

Although the question directs you to write about two different aspects of the passage, the essay itself should be unified. That is, a good essay should not consist of, say, two disparate paragraphs, one exclusively devoted to structure and another to language. Rather, the essay should include material that shows the interrelationship of structure and language in the passage and how those elements contribute to the meaning and effect of the passage. This might be covered in a separate paragraph, or it could be woven into the overall fabric of the essay. 

Before you begin to write, read the passage at least twice: once for an overview and once as you write your analysis. You may notice early on that the opening paragraph contains generalizations about Westerners’ concepts of science and progress. Then the author contrasts the Western view of science and progress with the Eastern view. Immediately, you see that the author, by using the first-person pronoun (as in “many of us”) is speaking from the perspective of an Easterner. Consequently, his discussion of Eastern views is apt to come across as more well-informed, more authoritative, perhaps more personal. 

To support his position, the author gives an extended example—the city of Shiraz—to illustrate just how different the East is from the West. The description and vivid images of Shiraz memorably convey the idea that the “spiritual way of life” has a side to it that many Westerners don’t know about. This is the heart of the passage. The use of quotation marks around “romantic” and “city of poetry” is meant to point out the discrepancy between the idealized and real versions of Shiraz. 

Nearing the end, the author reiterates his initial contrast between West and East, with emphasis on the East. The last paragraph offers a generalized statement about conditions in Asia and Africa, reminding the reader of the contrast made at the very beginning of the passage. Tying the end to the beginning of the passage creates a sense of unity—a desirable feature in any piece of writing.

AP English Language and Composition: Sample Argument Question

The following paragraph is adapted from Mirror for Man, a book written by anthropologist Clyde Kluckhorn in the middle of the twentieth century. Read the passage carefully. Then, write an essay that examines the extent to which the author’s characterization of the United States holds true today. Use appropriate evidence to support your argument. 

Sample Question Instructions: 

  • Respond to the prompt with a thesis that may establish a line of reasoning. 
  • Select and use evidence to develop and support the line of reasoning. 
  • Explain the relationship between the evidence and the thesis. 
  • Demonstrate an understanding of the rhetorical situation. 

Technology is valued as the very basis of the capitalistic system. Possession of gadgets is esteemed as a mark of success to the extent that persons are judged not by the integrity of their characters or by the originality of their minds but by what they seem to be—so far as can be measured by their wealth or by the variety and material goods which they display. “Success” is measured by their investments, homes, and lifestyles— not by their number of mistresses as in some cultures.

How to Answer the AP English Language and Composition Argument Question

Whether you agree, disagree, or have mixed views on the content of the passage, your job is to write a convincing argument that expresses your opinion. Initially, the word argument may suggest conflict or confrontation. But rest assured that your essay need not be combative. Rather, make it a calmly-reasoned explanation of your opinion on a debatable subject. Your goal is to persuade the reader that your opinion, supported by examples, facts, and other appropriate evidence, is correct. 

If you have strong feelings about the topic, of course you should state them in your essay. But express them in calm, rational language. Be mindful that the essay should not be an emotional rant for or against the issue. 

Consider first whether you agree with Kluckhorn’s definition of “success.” Is it, as Kluckhorn asserts, measured by income and material possessions? Or do you think that a more accurate standard of success in today’s America should be determined by less tangible criteria—things such as happiness or self-respect? Or do you stand somewhere in between those two extremes? 

The actual position you take on the issue is less crucial than your ability to support it fully by drawing from your knowledge, background, experience, or observation. Regardless of your position, be sure to include more than one example. An argument that relies on a single example, however compelling, will fall flat. 

In the prompt, Kluckhorn’s notion of success seems to refer broadly to American society. Resist responding in kind. That is, a short essay shouldn’t focus on the whole of society but only on an identifiable segment—perhaps college-educated professionals or urban, blue- collar Americans. The point is that a narrowly focused essay on a limited topic will always turn out better than one that tries to cover too much ground in just a few paragraphs.

AP Biology Resources

  • About the AP Biology Exam
  • Top AP Biology Exam Strategies
  • Top 5 Study Topics and Tips for the AP Biology Exam
  • AP Biology Short Free-Response Questions
  • AP Biology Long Free-Response Questions

AP Psychology Resources

  • What’s Tested on the AP Psychology Exam?
  • Top 5 Study Tips for the AP Psychology Exam
  • AP Psychology Key Terms
  • Top AP Psychology Exam Multiple-Choice Question Tips
  • Top AP Psychology Exam Free Response Questions Tips
  • AP Psychology Sample Free Response Question

AP English Language and Composition Resources

  • What’s Tested on the AP English Language and Composition Exam?
  • Top 5 Tips for the AP English Language and Composition Exam
  • Top Reading Techniques for the AP English Language and Composition Exam
  • How to Answer the AP English Language and Composition Essay Questions 
  • AP English Language and Composition Exam Sample Essay Questions
  • AP English Language and Composition Exam Multiple-Choice Questions

AP Human Geography Resources

  • What’s Tested On the AP Human Geography Exam?
  • AP Human Geography FAQs
  • AP Human Geography Question Types and Strategies
  • Top 5 Study Tips for the AP Human Geography Exam

FOLLOW ALONG ON SOCIAL

What are your chances of acceptance?

Calculate for all schools, your chance of acceptance.

Duke University

Your chancing factors

Extracurriculars.

ap lang rhetorical essay prompts

How to Write the AP Lang Argument Essay + Examples

What’s covered:, what is the ap language argument essay, tips for writing the ap language argument essay, ap english language argument essay examples, how will ap scores impact my college chances.

In 2023, over 550,148 students across the U.S. took the AP English Language and Composition Exam, and 65.2% scored higher than a 3. The AP English Language Exam tests your ability to analyze a piece of writing, synthesize information, write a rhetorical essay, and create a cohesive argument. In this post, we’ll be discussing the best way to approach the argumentative essay section of the test, and we’ll give you tips and tricks so you can write a great essay.

The AP English Language Exam as of 2023 is structured as follows:

Section 1: 45 multiple choice questions to be completed in an hour. This portion counts for 45% of your score. This section requires students to analyze a piece of literature. The questions ask about its content and/or what could be edited within the passage.

Section 2: Three free response questions to be completed in the remaining two hours and 15 minutes. This section counts for 55% of your score. These essay questions include the synthesis essay, the rhetorical essay, and the argumentative essay.

  • Synthesis essay: Read 6-7 sources and create an argument using at least three of the sources.
  • Rhetorical analysis essay: Describe how a piece of writing evokes meaning and symbolism.
  • Argumentative essay: Pick a side of a debate and create an argument based on evidence. In this essay, you should develop a logical argument in support of or against the given statement and provide ample evidence that supports your conclusion. Typically, a five paragraph format is great for this type of writing. This essay is scored holistically from 1 to 9 points.

Do you want more information on the structure of the full exam? Take a look at our in-depth overview of the AP Language and Composition Exam .

Although the AP Language Argument may seem daunting at first, once you understand how the essay should be structured, it will be a lot easier to create cohesive arguments.

Below are some tips to help you as you write the essay.

1. Organize your essay before writing

Instead of jumping right into your essay, plan out what you will say beforehand. It’s easiest to make a list of your arguments and write out what facts or evidence you will use to support each argument. In your outline, you can determine the best order for your arguments, especially if they build on each other or are chronological. Having a well-organized essay is crucial for success.

2. Pick one side of the argument, but acknowledge the other side

When you write the essay, it’s best if you pick one side of the debate and stick with it for the entire essay. All your evidence should be in support of that one side. However, in your introductory paragraph, as you introduce the debate, be sure to mention any merit the arguments of the other side has. This can make the essay a bit more nuanced and show that you did consider both sides before determining which one was better. Often, acknowledging another viewpoint then refuting it can make your essay stronger.

3. Provide evidence to support your claims

The AP readers will be looking for examples and evidence to support your argument. This doesn’t mean that you need to memorize a bunch of random facts before the exam. This just means that you should be able to provide concrete examples in support of your argument.

For example, if the essay topic is about whether the role of the media in society has been detrimental or not, and you argue that it has been, you may talk about the phenomenon of “fake news” during the 2016 presidential election.

AP readers are not looking for perfect examples, but they are looking to see if you can provide enough evidence to back your claim and make it easily understood.

4. Create a strong thesis statement

The thesis statement will set up your entire essay, so it’s important that it is focused and specific, and that it allows for the reader to understand your body paragraphs. Make sure your thesis statement is the very last sentence of your introductory paragraph. In this sentence, list out the key points you will be making in the essay in the same order that you will be writing them. Each new point you mention in your thesis should start a paragraph in your essay.

Below is a prompt and sample student essay from the May 2019 exam . We’ll look at what the student did well in their writing and where they could improve.

Prompt: “The term “overrated” is often used to diminish concepts, places, roles, etc. that the speaker believes do not deserve the prestige they commonly enjoy; for example, many writers have argued that success is overrated, a character in a novel by Anthony Burgess famously describes Rome as a “vastly overrated city,” and Queen Rania of Jordan herself has asserted that “[b]eing queen is overrated.”

Select a concept, place, role, etc. to which you believe that the term “overrated” should be applied. Then, write a well-developed essay in which you explain your judgment. Use appropriate evidence from your reading, experience, or observations to support your argument.

Sample Student Essay #1:

[1] Competition is “overrated.” The notion of motivation between peers has evolved into a source of unnecessary stress and even lack of morals. Whether it be in an academic environment or in the industry, this new idea of competition is harmful to those competing and those around them.

[2] Back in elementary school, competition was rather friendly. It could have been who could do the most pushups or who could get the most imaginary points in a classroom for a prize. If you couldn’t do the most pushups or win that smelly sticker, you would go home and improve yourself – there would be no strong feelings towards anyone, you would just focus on making yourself a better version of yourself. Then as high school rolled around, suddenly applying for college doesn’t seem so far away –GPA seems to be that one stat that defines you – extracurriculars seem to shape you – test scores seem to categorize you. Sleepless nights, studying for the next day’s exam, seem to become more and more frequent. Floating duck syndrome seems to surround you (FDS is where a competitive student pretends to not work hard but is furiously studying beneath the surface just like how a duck furiously kicks to stay afloat). All of your competitors appear to hope you fail – but in the end what do you and your competitor’s gain? Getting one extra point on the test? Does that self-satisfaction compensate for the tremendous amounts of acquired stress? This new type of “competition” is overrated – it serves nothing except a never-ending source of anxiety and seeks to weaken friendships and solidarity as a whole in the school setting.

[3] A similar idea of “competition” can be applied to business. On the most fundamental level, competition serves to be a beneficial regulator of prices and business models for both the business themselves and consumers. However, as businesses grew increasingly greedy and desperate, companies resorted to immoral tactics that only hurt their reputations and consumers as a whole. Whether it be McDonald’s coupons that force you to buy more food or tech companies like Apple intentionally slowing down your iPhone after 3 years to force you to upgrade to the newest device, consumers suffer and in turn speak down upon these companies. Similar to the evolved form of competition in school, this overrated form causes pain for all parties and has since diverged from the encouraging nature that the principle of competition was “founded” on.

The AP score for this essay was a 4/6, meaning that it captured the main purpose of the essay but there were still substantial parts missing. In this essay, the writer did a good job organizing the sections and making sure that their writing was in order according to the thesis statement. The essay first discusses how competition is harmful in elementary school and then discusses this topic in the context of business. This follows the chronological order of somebody’s life and flows nicely.

The arguments in this essay are problematic, as they do not provide enough examples of how exactly competition is overrated. The essay discusses the context in which competition is overrated but does not go far enough in explaining how this connects to the prompt.

In the first example, school stress is used to explain how competition manifests. This is a good starting point, but it does not talk about why competition is overrated; it simply mentions that competition can be unhealthy. The last sentence of that paragraph is the main point of the argument and should be expanded to discuss how the anxiety of school is overrated later on in life. 

In the second example, the writer discusses how competition can lead to harmful business practices, but again, this doesn’t reflect the reason this would be overrated. Is competition really overrated because Apple and McDonald’s force you to buy new products? This example could’ve been taken one step farther. Instead of explaining why business structures—such as monopolies—harm competition, the author should discuss how those particular structures are overrated.

Additionally, the examples the writer used lack detail. A stronger essay would’ve provided more in-depth examples. This essay seemed to mention examples only in passing without using them to defend the argument.

It should also be noted that the structure of the essay is incomplete. The introduction only has a thesis statement and no additional context. Also, there is no conclusion paragraph that sums up the essay. These missing components result in a 4/6.

Now let’s go through the prompt for a sample essay from the May 2022 exam . The prompt is as follows:

Colin Powell, a four-star general and former United States Secretary of State, wrote in his 1995 autobiography: “[W]e do not have the luxury of collecting information indefinitely. At some point, before we can have every possible fact in hand, we have to decide. The key is not to make quick decisions, but to make timely decisions.”

Write an essay that argues your position on the extent to which Powell’s claim about making decisions is valid. 

In your response you should do the following:

  • Respond to the prompt with a thesis that presents a defensible position. 
  • Provide evidence to support your line of reasoning. 
  • Explain how the evidence supports your line of reasoning. 
  • Use appropriate grammar and punctuation in communicating your argument.

Sample Student Essay #2:

Colin Powell, who was a four star general and a former United States Secretary of State. He wrote an autobiography and had made a claim about making decisions. In my personal opinion, Powell’s claim is true to full extent and shows an extremely valuable piece of advice that we do not consider when we make decisions.

Powell stated, “before we can have every possible fact in hand we have to decide…. but to make it a timely decision” (1995). With this statement Powell is telling the audience of his autobiography that it does not necessarily matter how many facts you have, and how many things you know. Being able to have access to everything possible takes a great amount of time and we don’t always have all of the time in the world. A decision has to be made with what you know, waiting for something else to come in while trying to make a decision whether that other fact is good or bad you already have a good amount of things that you know. Everyone’s time is valuable, including yours. At the end of the day the decision will have to be made and that is why it should be made in a “timely” manner.

This response was graded for a score of 2/6. Let’s break down the score to smaller points that signify where the student fell short.

The thesis in this essay is clearly outlined at the end of the first paragraph. The student states their agreement with Powell’s claim and frames the rest of their essay around this stance. The success in scoring here lies in the clear communication of the thesis and the direction the argument will take. It’s important to make the thesis statement concise, specific, and arguable, which the student has successfully done.

While the student did attempt to provide evidence to support their thesis, it’s clear that their explanation lacks specific detail and substance. They referenced Powell’s statement, but did not delve into how this statement has proven true in specific instances, and did not provide examples that could bring the argument to life.

Commentary is an essential part of this section’s score. It means explaining the significance of the evidence and connecting it back to the thesis. Unfortunately, the student’s commentary here is too vague and does not effectively elaborate on how the evidence supports their argument.

To improve, the student could use more concrete examples to demonstrate their point and discuss how each piece of evidence supports their thesis. For instance, they could discuss specific moments in Powell’s career where making a timely decision was more valuable than waiting for all possible facts. This would help illustrate the argument in a more engaging, understandable way.

A high score in the “sophistication” category of the grading rubric is given for demonstrating a complex understanding of the rhetorical situation (purpose, audience, context, etc.), making effective rhetorical choices, or establishing a line of reasoning. Here, the student’s response lacks complexity and sophistication. They’ve simply agreed with Powell’s claim and made a few general statements without providing a deeper analysis or effectively considering the rhetorical situation.

To increase sophistication, the student could explore possible counterarguments or complexities within Powell’s claim. They could discuss potential drawbacks of making decisions without all possible facts, or examine situations where timely decisions might not yield the best results. By acknowledging and refuting these potential counterarguments, they could add more depth to their analysis and showcase their understanding of the complexities involved in decision-making.

The student could also analyze why Powell, given his background and experiences, might have come to such a conclusion, thus providing more context and showing an understanding of the rhetorical situation.

Remember, sophistication in argumentation isn’t about using fancy words or complicated sentences. It’s about showing that you understand the complexity of the issue at hand and that you’re able to make thoughtful, nuanced arguments. Sophistication shows that you can think critically about the topic and make connections that aren’t immediately obvious.

Now that you’ve looked at an example essay and some tips for the argumentative essay, you know how to better prepare for the AP English Language and Composition Exam.

While your AP scores don’t usually impact your admissions chances , colleges do care a lot about your course rigor. So, taking as many APs as you can will certainly boost your chances! AP scores can be a way for high-performing students to set themselves apart, particularly when applying to prestigious universities. Through the process of self-reporting scores , you can show your hard work and intelligence to admissions counselors.

That said, the main benefit of scoring high on AP exams comes once you land at your dream school, as high scores can allow you to “test out” of entry-level requirements, often called GE requirements or distribution requirements. This will save you time and money.

To understand how your course rigor stacks up, check out CollegeVine’s free chancing engine . This resource takes your course rigor, test scores, extracurriculars, and more, to determine your chances of getting into over 1600 colleges across the country!

Related CollegeVine Blog Posts

ap lang rhetorical essay prompts

Get in touch with us

Are you sure you want to logout?

Study abroad.

bannerAd

AP Lang Rhetorical Analysis Essay

AP language exams are held for numerous subjects. Students have to choose a specific subject and get higher scores. The higher the score, the higher the chance of pursuing the best 700 colleges or universities overall in the world, including the United States and Canada.

When it comes to AP English language, it involves a section called rhetorical analysis essay. This is a part of three free-response essays that have to be answered within 2 hours and 15 minutes from the overall 3 hours 15 minutes exam. 

parallel

If you are taking the AP Lang exam this year, guidance on how to answer this part will be useful. This article is specially curated to help you score the best. Read on to learn more about the AP language rhetorical analysis essay and get an idea of how to prepare for the associated exam successfully.

What is the AP Lang Rhetorical Analysis Essay ?

AP elaborated that the Advanced Placement is the exam conducted by the College Board in the United States of America. The exam is generally offered at the high school level and helps students to pursue higher education at the university level. The exam holds two sections: MCQs and Rhetorical Essay. MCQ sections are for 1 hour, and the rest of the time is for the essay section. The free-response essay holds three essays: rhetorical analysis essay, synthesis essay, and argumentative essay. 

parallel

  • The rhetorical analysis essay in the AP Lang exam involves students having to discuss how the authors’ contribution to the passage gives a theme or meaning. 
  • A synthesis essay involves students creating arguments on the passage or piece of information delivered to them.
  • An argumentative essay requires students to pick a side ‘for or against ‘ for an argument or debate.

Since we are here to discuss rhetorical analysis essays in AP language, you must know the essay is added to test students’ ability to analyze and interpret the deeper meaning in the provided passage. Through rhetorical essays, the examiner examines how students connect with the author’s style of writing and syntax within 40 minutes. Some students may face challenges while dealing with this essay because it requires a better understanding of rhetorical strategies and the method to apply them.

Tips to Write a Rhetorical Analysis Essay AP Lang ?

Since a rhetorical essay is quite tricky in comparison to other essays, it requires certain tips for a better approach to answering. Let us explore the method of writing a rhetorical essay for the AP Lang exam that helps you score well.

parallel

  • Outline Essay Prior to Writing

Not only good content but providing a readable structure is an important part of the rhetorical analysis essay AP Lang . First, you must read the passage thoroughly and develop a brief outline or key points before writing the essay. This helps you write with respect to the chronology of the given passage and maintain the flow of writing.

  • Understand Rhetorical Strategies

You aren’t alone in thinking about where to start writing a rhetorical essay. Many students face the same and are required to learn and implement rhetorical strategies. First, understand what rhetoric actually means. It refers to language sensibly chosen and structured for an impressive effect on the audience. This involves persuasive appeal, logical fallacies, and syntax such as anaphora, anthesis, parallelism, and so on. There is a vast range of elements that you can assess in the provided essay and develop a strong grasp with consistent practice.

parallel

  • Make your Essay Well-Structured

Sometimes, students understand the rhetorical strategies but still get confused about where to initiate. It is recommended to start with an introduction that delivers the purpose of your writing. In the last introductory line, you must talk about the rhetorical strategies you will discuss in the piece. However, there are so many styles, syntax, and tones. You must be specific while listing them and then move to develop a body paragraph. 

Now, you have to collect all the rhetorical strategies you mentioned in the introduction to discuss your point of view chronologically. Be specific while discussing strategy, as only the crucial ones among them must be discussed. Never forget to cite the line from the original passage. Your write-up must maintain the flow and should include relativity among the paragraphs. End the rhetorical essay by summarising key points.

parallel

  • Never Forget to Explain your Examples

Do not just state the examples or deliver statements like ‘this is an example of pathos or logos.’ It is advisable to explain the example you have listed in context to the rhetorical elements you have mentioned and how it aids the author in their viewpoint. Stay detailed yet precise while writing the rhetorical analysis essay AP Lang .

AP Lang Rhetorical Analysis Essay Rubric

Did you know that the rhetorical analysis essay AP Lang is graded into three rubric categories? Students must pay heed to the categories, as the examiner seeks specific things in each of them. Also, learn about some dos and don’ts to score well.

parallel

When it comes to grading the thesis of the rhetorical analysis essay AP Lang , there is nothing nebulous. Either you get one point, or you lose one. Thesis points help you get close to higher scores, and thus, you must be mindful of the following points.

parallel

  • Developing an argument states that your interpretation leads to a risk of disagreement. Thus, your thesis statements must be in context with the author’s rhetorical choice. 
  • If you create a phrase in your mind initiating with ‘I think that..’, your phrase should not go in a negative argumentative direction.
  • Never provide a thesis with summaries but not an argument.
  • Try not to provide the thesis with repeated prompts.

Evidence is provided to prove the argumentative context in the thesis. This rubric category is graded from 0 to 4 points. To score higher, you must follow the points below.

  • You must aim to provide multiple types of evidence in your argumentative thesis.
  • Each statement you provide must be backed up with evidence in context to the text or arguments about the author’s theoretical choices.
  • Add more and more evidence, which must be specific.
  • Examine whether your evidence is linked with your overarching argument.
  • Deliver your interpretation and never rely on just quotes or phrases. 
  • Steer clear of generalization for text or author
  • Avoid quotes that speak for themselves. You must elaborate on the evidence you provided.
  • Sophistication

According to The College Board, this category holds 0 to 1 points. The higher grading in rhetorical analysis essays indicates the delivery of the sophistication of thoughts or a complex understanding of rhetorical elements. In this, you have to mainly focus on the number of semicolons you use and not the fancy terms. Here is what you need to follow:

  • Focus on delivering the right connection between the thesis and your evidence.
  • Create a brief framework and then proceed with writing the essay in the right flow to stay precise and clear in your piece.
  • Do not include arguments that you won’t be able to provide evidence for.
  • Ignore complex or fancy words or phrases that are hard to follow.

Ways to Improve Your Rhetorical Analysis Essay AP Lang

Now that you know what a rhetorical analysis essay is and how it is developed, you must be thinking about how tricky it is to understand the passage. Many find themselves in a completely blank position when it comes to where to begin during the exam. They find the rhetorical analysis essay AP Lang quite challenging to deal with and understand the author’s perspective and viewpoint. Practicing rhetorical strategies does not help if you do not have the right approach, which only comes from the field experts and their guidance.  Turito offers a platform where you get mentors to teach you all the concepts and approaches you apply in writing effective rhetorical essays in less time. Their course helps you get higher scores, which you can use to get college admission to your favorite college and country. Hey, you ambitious one, what are you waiting for? Contact us now and give your dreams a flight!

Frequently Asked Questions

What courses do turito offer.

Turito offers several courses to help you prepare. Here are some of them for your reference. 1. Foundation Course 2. IIT JEE 3. NEET 4. Study Abroad 5. Universal Program 6. PSAT 7. AP 8. IELTS

Can I ask queries on Turito?

Yes absolutely! Turito offers a platform where you can ask your concerns and queries. You will get answers from the field expert that can let your worries fade away.

Is Turito genuine?

Turito is students’ most trusted learning platform, among others. Turito believes in transparency and thus provides genuine feedback from students and parents on the official website. At Turito, we offer guaranteed success for hardworking students. Thus, there isn’t any negative feedback to hide. If you have any queries, you can write to us at [email protected]. Our experts will connect and assist you in no time.

AP Lang Rhetorical Analysis Essay

Relevant Articles

AP Lang Synthesis Essay

Steps to Draft AP Lang Synthesis Essay

The synthesis essay AP lang is part of the AP …

Steps to Draft AP Lang Synthesis Essay Read More »

AP Calculus AB Exam

Strategies for Success: A Complete Overview of the AP Calculus AB Exam

The AP Calculus AB Exam serves as an important element …

Strategies for Success: A Complete Overview of the AP Calculus AB Exam Read More »

Digital AP Exams

Preparing for Success: A Guide to the Features of Digital AP Exams

Digital  Advanced Placement (AP) exams bring about a huge change …

Preparing for Success: A Guide to the Features of Digital AP Exams Read More »

card img

With Turito Study Abroad

card img

Get an Expert Advice from Turito

card img

With Turito CAP.

card img

With Turito Coding.

card img

With Turito RoboNinja

card img

1-on-1 tutoring for the undivided attention

PrepScholar

Choose Your Test

Sat / act prep online guides and tips, how to write a perfect synthesis essay for the ap language exam.

author image

Advanced Placement (AP)

body-pencil-sharpen-notebook-1

If you're planning to take the AP Language (or AP Lang) exam , you might already know that 55% of your overall exam score will be based on three essays. The first of the three essays you'll have to write on the AP Language exam is called the "synthesis essay." If you want to earn full points on this portion of the AP Lang Exam, you need to know what a synthesis essay is and what skills are assessed by the AP Lang synthesis essay.

In this article, we'll explain the different aspects of the AP Lang synthesis essay, including what skills you need to demonstrate in your synthesis essay response in order to achieve a good score. We'll also give you a full breakdown of a real AP Lang Synthesis Essay prompt, provide an analysis of an AP Lang synthesis essay example, and give you four tips for how to write a synthesis essay.

Let's get started by taking a closer look at how the AP Lang synthesis essay works!

Synthesis Essay AP Lang: What It Is and How It Works

The AP Lang synthesis essay is the first of three essays included in the Free Response section of the AP Lang exam.

The AP Lang synthesis essay portion of the Free Response section lasts for one hour total . This hour consists of a recommended 15 minute reading period and a 40 minute writing period. Keep in mind that these time allotments are merely recommendations, and that exam takers can parse out the allotted 60 minutes to complete the synthesis essay however they choose.

Now, here's what the structure of the AP Lang synthesis essay looks like. The exam presents six to seven sources that are organized around a specific topic (like alternative energy or eminent domain, which are both past synthesis exam topics).

Of these six to seven sources, at least two are visual , including at least one quantitative source (like a graph or pie chart, for example). The remaining four to five sources are print text-based, and each one contains approximately 500 words.

In addition to six to seven sources, the AP Lang exam provides a written prompt that consists of three paragraphs. The prompt will briefly explain the essay topic, then present a claim that students will respond to in an essay that synthesizes material from at least three of the sources provided.

Here's an example prompt provided by the College Board:

Directions : The following prompt is based on the accompanying six sources.

This question requires you to integrate a variety of sources into a coherent, well-written essay. Refer to the sources to support your position; avoid mere paraphrase or summary. Your argument should be central; the sources should support this argument .

Remember to attribute both direct and indirect citations.

Introduction

Television has been influential in United States presidential elections since the 1960's. But just what is this influence, and how has it affected who is elected? Has it made elections fairer and more accessible, or has it moved candidates from pursuing issues to pursuing image?

Read the following sources (including any introductory information) carefully. Then, in an essay that synthesizes at least three of the sources for support, take a position that defends, challenges, or qualifies the claim that television has had a positive impact on presidential elections.

Refer to the sources as Source A, Source B, etc.; titles are included for your convenience.

Source A (Campbell) Source B (Hart and Triece) Source C (Menand) Source D (Chart) Source E (Ranney) Source F (Koppel)

Like we mentioned earlier, this prompt gives you a topic — which it briefly explains — then asks you to take a position. In this case, you'll have to choose a stance on whether television has positively or negatively affected U.S. elections. You're also given six sources to evaluate and use in your response. Now that you have everything you need, now your job is to write an amazing synthesis essay.

But what does "synthesize" mean, exactly? According to the CollegeBoard, when an essay prompt asks you to synthesize, it means that you should "combine different perspectives from sources to form a support of a coherent position" in writing. In other words, a synthesis essay asks you to state your claim on a topic, then highlight the relationships between several sources that support your claim on that topic. Additionally, you'll need to cite specific evidence from your sources to prove your point.

The synthesis essay counts for six of the total points on the AP Lang exam . Students can receive 0-1 points for writing a thesis statement in the essay, 0-4 based on incorporation of evidence and commentary, and 0-1 points based on sophistication of thought and demonstrated complex understanding of the topic.

You'll be evaluated based on how effectively you do the following in your AP Lang synthesis essay:

Write a thesis that responds to the exam prompt with a defensible position

Provide specific evidence that to support all claims in your line of reasoning from at least three of the sources provided, and clearly and consistently explain how the evidence you include supports your line of reasoning

Demonstrate sophistication of thought by either crafting a thoughtful argument, situating the argument in a broader context, explaining the limitations of an argument

Make rhetorical choices that strengthen your argument and/or employ a vivid and persuasive style throughout your essay.

If your synthesis essay meets the criteria above, then there's a good chance you'll score well on this portion of the AP Lang exam!

If you're looking for even more information on scoring, the College Board has posted the AP Lang Free Response grading rubric on its website. ( You can find it here. ) We recommend taking a close look at it since it includes additional details about the synthesis essay scoring.

body-chisel-break-apart

Don't be intimidated...we're going to teach you how to break down even the hardest AP synthesis essay prompt.

Full Breakdown of a Real AP Lang Synthesis Essay Prompt

In this section, we'll teach you how to analyze and respond to a synthesis essay prompt in five easy steps, including suggested time frames for each step of the process.

Step 1: Analyze the Prompt

The very first thing to do when the clock starts running is read and analyze the prompt. To demonstrate how to do this, we'll look at the sample AP Lang synthesis essay prompt below. This prompt comes straight from the 2018 AP Lang exam:

Eminent domain is the power governments have to acquire property from private owners for public use. The rationale behind eminent domain is that governments have greater legal authority over lands within their dominion than do private owners. Eminent domain has been instituted in one way or another throughout the world for hundreds of years.

Carefully read the following six sources, including the introductory information for each source. Then synthesize material from at least three of the sources and incorporate it into a coherent, well-developed essay that defends, challenges, or qualifies the notion that eminent domain is productive and beneficial.

Your argument should be the focus of your essay. Use the sources to develop your argument and explain the reasoning for it. Avoid merely summarizing the sources. Indicate clearly which sources you are drawing from, whether through direct quotation, paraphrase, or summary. You may cite the sources as Source A, Source B, etc., or by using the descriptions in parentheses.

On first read, you might be nervous about how to answer this prompt...especially if you don't know what eminent domain is! But if you break the prompt down into chunks, you'll be able to figure out what the prompt is asking you to do in no time flat.

To get a full understanding of what this prompt wants you to do, you need to identify the most important details in this prompt, paragraph by paragraph. Here's what each paragraph is asking you to do:

  • Paragraph 1: The prompt presents and briefly explains the topic that you'll be writing your synthesis essay about. That topic is the concept of eminent domain.
  • Paragraph 2: The prompt presents a specific claim about the concept of eminent domain in this paragraph: Eminent domain is productive and beneficial. This paragraph instructs you to decide whether you want to defend, challenge, or qualify that claim in your synthesis essay , and use material from at least three of the sources provided in order to do so.
  • Paragraph 3: In the last paragraph of the prompt, the exam gives you clear instructions about how to approach writing your synthesis essay . First, make your argument the focus of the essay. Second, use material from at least three of the sources to develop and explain your argument. Third, provide commentary on the material you include, and provide proper citations when you incorporate quotations, paraphrases, or summaries from the sources provided.

So basically, you'll have to agree with, disagree with, or qualify the claim stated in the prompt, then use at least three sources substantiate your answer. Since you probably don't know much about eminent domain, you'll probably decide on your position after you read the provided sources.

To make good use of your time on the exam, you should spend around 2 minutes reading the prompt and making note of what it's asking you to do. That will leave you plenty of time to read the sources provided, which is the next step to writing a synthesis essay.

Step 2: Read the Sources Carefully

After you closely read the prompt and make note of the most important details, you need to read all of the sources provided. It's tempting to skip one or two sources to save time--but we recommend you don't do this. That's because you'll need a thorough understanding of the topic before you can accurately address the prompt!

For the sample exam prompt included above, there are six sources provided. We're not going to include all of the sources in this article, but you can view the six sources from this question on the 2018 AP Lang exam here . The sources include five print-text sources and one visual source, which is a cartoon.

As you read the sources, it's important to read quickly and carefully. Don't rush! Keep your pencil in hand to quickly mark important passages that you might want to use as evidence in your synthesis. While you're reading the sources and marking passages, you want to think about how the information you're reading influences your stance on the issue (in this case, eminent domain).

When you finish reading, take a few seconds to summarize, in a phrase or sentence, whether the source defends, challenges, or qualifies whether eminent domain is beneficial (which is the claim in the prompt) . Though it might not feel like you have time for this, it's important to give yourself these notes about each source so you know how you can use each one as evidence in your essay.

Here's what we mean: say you want to challenge the idea that eminent domain is useful. If you've jotted down notes about each source and what it's saying, it will be easier for you to pull the relevant information into your outline and your essay.

So how much time should you spend reading the provided sources? The AP Lang exam recommends taking 15 minutes to read the sources . If you spend around two of those minutes reading and breaking down the essay prompt, it makes sense to spend the remaining 13 minutes reading and annotating the sources.

If you finish reading and annotating early, you can always move on to drafting your synthesis essay. But make sure you're taking your time and reading carefully! It's better to use a little extra time reading and understanding the sources now so that you don't have to go back and re-read the sources later.

body-weightlifting-lift-strong

A strong thesis will do a lot of heavy lifting in your essay. (See what we did there?)

Step 3: Write a Strong Thesis Statement

After you've analyzed the prompt and thoroughly read the sources, the next thing you need to do in order to write a good synthesis essay is write a strong thesis statement .

The great news about writing a thesis statement for this synthesis essay is that you have all the tools you need to do it at your fingertips. All you have to do in order to write your thesis statement is decide what your stance is in relationship to the topic provided.

In the example prompt provided earlier, you're essentially given three choices for how to frame your thesis statement: you can either defend, challenge, or qualify a claim that's been provided by the prompt, that eminent domain is productive and beneficial . Here's what that means for each option:

If you choose to defend the claim, your job will be to prove that the claim is correct . In this case, you'll have to show that eminent domain is a good thing.

If you choose to challenge the claim, you'll argue that the claim is incorrect. In other words, you'll argue that eminent domain isn't productive or beneficial.

If you choose to qualify, that means you'll agree with part of the claim, but disagree with another part of the claim. For instance, you may argue that eminent domain can be a productive tool for governments, but it's not beneficial for property owners. Or maybe you argue that eminent domain is useful in certain circumstances, but not in others.

When you decide whether you want your synthesis essay to defend, challenge, or qualify that claim, you need to convey that stance clearly in your thesis statement. You want to avoid simply restating the claim provided in the prompt, summarizing the issue without making a coherent claim, or writing a thesis that doesn't respond to the prompt.

Here's an example of a thesis statement that received full points on the eminent domain synthesis essay:

Although eminent domain can be misused to benefit private interests at the expense of citizens, it is a vital tool of any government that intends to have any influence on the land it governs beyond that of written law.

This thesis statement received full points because it states a defensible position and establishes a line of reasoning on the issue of eminent domain. It states the author's position (that some parts of eminent domain are good, but others are bad), then goes on to explain why the author thinks that (it's good because it allows the government to do its job, but it's bad because the government can misuse its power.)

Because this example thesis statement states a defensible position and establishes a line of reasoning, it can be elaborated upon in the body of the essay through sub-claims, supporting evidence, and commentary. And a solid argument is key to getting a six on your synthesis essay for AP Lang!

Looking for help studying for your AP exam?

Our one-on-one online AP tutoring services can help you prepare for your AP exams. Get matched with a top tutor who got a high score on the exam you're studying for!

Get a 5 On Your AP Exam

Step 4: Create a Bare-Bones Essay Outline

Once you've got your thesis statement drafted, you have the foundation you need to develop a bare bones outline for your synthesis essay. Developing an outline might seem like it's a waste of your precious time, but if you develop your outline well, it will actually save you time when you start writing your essay.

With that in mind, we recommend spending 5 to 10 minutes outlining your synthesis essay . If you use a bare-bones outline like the one below, labeling each piece of content that you need to include in your essay draft, you should be able to develop out the most important pieces of the synthesis before you even draft the actual essay.

To help you see how this can work on test day, we've created a sample outline for you. You can even memorize this outline to help you out on test day! In the outline below, you'll find places to fill in a thesis statement, body paragraph topic sentences, evidence from the sources provided, and commentary :

  • Present the context surrounding the essay topic in a couple of sentences (this is a good place to use what you learned about the major opinions or controversies about the topic from reading your sources).
  • Write a straightforward, clear, and concise thesis statement that presents your stance on the topic
  • Topic sentence presenting first supporting point or claim
  • Evidence #1
  • Commentary on Evidence #1
  • Evidence #2 (if needed)
  • Commentary on Evidence #2 (if needed)
  • Topic sentence presenting second supporting point or claim
  • Topic sentence presenting three supporting point or claim
  • Sums up the main line of reasoning that you developed and defended throughout the essay
  • Reiterates the thesis statement

Taking the time to develop these crucial pieces of the synthesis in a bare-bones outline will give you a map for your final essay. Once you have a map, writing the essay will be much easier.

Step 5: Draft Your Essay Response

The great thing about taking a few minutes to develop an outline is that you can develop it out into your essay draft. After you take about 5 to 10 minutes to outline your synthesis essay, you can use the remaining 30 to 35 minutes to draft your essay and review it.

Since you'll outline your essay before you start drafting, writing the essay should be pretty straightforward. You'll already know how many paragraphs you're going to write, what the topic of each paragraph will be, and what quotations, paraphrases, or summaries you're going to include in each paragraph from the sources provided. You'll just have to fill in one of the most important parts of your synthesis—your commentary.

Commentaries are your explanation of why your evidence supports the argument you've outlined in your thesis. Your commentary is where you actually make your argument, which is why it's such a critical part of your synthesis essay.

When thinking about what to say in your commentary, remember one thing the AP Lang synthesis essay prompt specifies: don't just summarize the sources. Instead, as you provide commentary on the evidence you incorporate, you need to explain how that evidence supports or undermines your thesis statement . You should include commentary that offers a thoughtful or novel perspective on the evidence from your sources to develop your argument.

One very important thing to remember as you draft out your essay is to cite your sources. The AP Lang exam synthesis essay prompt indicates that you can use generic labels for the sources provided (e.g. "Source 1," "Source 2," "Source 3," etc.). The exam prompt will indicate which label corresponds with which source, so you'll need to make sure you pay attention and cite sources accurately. You can cite your sources in the sentence where you introduce a quote, summary, or paraphrase, or you can use a parenthetical citation. Citing your sources affects your score on the synthesis essay, so remembering to do this is important.

body-green-arrow-down

Keep reading for a real-life example of a great AP synthesis essay response!

Real-Life AP Synthesis Essay Example and Analysis

If you're still wondering how to write a synthesis essay, examples of real essays from past AP Lang exams can make things clearer. These real-life student AP synthesis essay responses can be great for helping you understand how to write a synthesis essay that will knock the graders' socks off .

While there are multiple essay examples online, we've chosen one to take a closer look at. We're going to give you a brief analysis of one of these example student synthesis essays from the 2019 AP Lang Exam below!

Example Synthesis Essay AP Lang Response

To get started, let's look at the official prompt for the 2019 synthesis essay:

In response to our society's increasing demand for energy, large-scale wind power has drawn attention from governments and consumers as a potential alternative to traditional materials that fuel our power grids, such as coal, oil, natural gas, water, or even newer sources such as nuclear or solar power. Yet the establishment of large-scale, commercial-grade wind farms is often the subject of controversy for a variety of reasons.

Carefully read the six sources, found on the AP English Language and Composition 2019 Exam (Question 1), including the introductory information for each source. Write an essay that synthesizes material from at least three of the sources and develops your position on the most important factors that an individual or agency should consider when deciding whether to establish a wind farm.

Source A (photo) Source B (Layton) Source C (Seltenrich) Source D (Brown) Source E (Rule) Source F (Molla)

In your response you should do the following:

  • Respond to the prompt with a thesis presents a defensible position.
  • Select and use evidence from at least 3 of the provided sources to support your line of reasoning. Indicate clearly the sources used through direct quotation, paraphrase, or summary. Sources may be cited as Source A, Source B, etc., or by using the description in parentheses.
  • Explain how the evidence supports your line of reasoning.
  • Use appropriate grammar and punctuation in communicating your argument.

Now that you know exactly what the prompt asked students to do on the 2019 AP Lang synthesis essay, here's an AP Lang synthesis essay example, written by a real student on the AP Lang exam in 2019:

[1] The situation has been known for years, and still very little is being done: alternative power is the only way to reliably power the changing world. The draw of power coming from industry and private life is overwhelming current sources of non-renewable power, and with dwindling supplies of fossil fuels, it is merely a matter of time before coal and gas fuel plants are no longer in operation. So one viable alternative is wind power. But as with all things, there are pros and cons. The main factors for power companies to consider when building wind farms are environmental boon, aesthetic, and economic factors.

[2] The environmental benefits of using wind power are well-known and proven. Wind power is, as qualified by Source B, undeniably clean and renewable. From their production requiring very little in the way of dangerous materials to their lack of fuel, besides that which occurs naturally, wind power is by far one of the least environmentally impactful sources of power available. In addition, wind power by way of gearbox and advanced blade materials, has the highest percentage of energy retention. According to Source F, wind power retains 1,164% of the energy put into the system – meaning that it increases the energy converted from fuel (wind) to electricity 10 times! No other method of electricity production is even half that efficient. The efficiency and clean nature of wind power are important to consider, especially because they contribute back to power companies economically.

[3] Economically, wind power is both a boon and a bone to electric companies and other users. For consumers, wind power is very cheap, leading to lower bills than from any other source. Consumers also get an indirect reimbursement by way of taxes (Source D). In one Texan town, McCamey, tax revenue increased 30% from a wind farm being erected in the town. This helps to finance improvements to the town. But, there is no doubt that wind power is also hurting the power companies. Although, as renewable power goes, wind is incredibly cheap, it is still significantly more expensive than fossil fuels. So, while it is helping to cut down on emissions, it costs electric companies more than traditional fossil fuel plants. While the general economic trend is positive, there are some setbacks which must be overcome before wind power can take over as truly more effective than fossil fuels.

[4] Aesthetics may be the greatest setback for power companies. Although there may be significant economic and environmental benefit to wind power, people will always fight to preserve pure, unspoiled land. Unfortunately, not much can be done to improve the visual aesthetics of the turbines. White paint is the most common choice because it "[is] associated with cleanliness." (Source E). But, this can make it stand out like a sore thumb, and make the gargantuan machines seem more out of place. The site can also not be altered because it affects generating capacity. Sound is almost worse of a concern because it interrupts personal productivity by interrupting people's sleep patterns. One thing for power companies to consider is working with turbine manufacturing to make the machines less aesthetically impactful, so as to garner greater public support.

[5] As with most things, wind power has no easy answer. It is the responsibility of the companies building them to weigh the benefits and the consequences. But, by balancing economics, efficiency, and aesthetics, power companies can create a solution which balances human impact with environmental preservation.

And that's an entire AP Lang synthesis essay example, written in response to a real AP Lang exam prompt! It's important to remember AP Lang exam synthesis essay prompts are always similarly structured and worded, and students often respond in around the same number of paragraphs as what you see in the example essay response above.

Next, let's analyze this example essay and talk about what it does effectively, where it could be improved upon, and what score past exam scorers awarded it.

To get started on an analysis of the sample synthesis essay, let's look at the scoring commentary provided by the College Board:

  • For development of thesis, the essay received 1 out of 1 possible points
  • For evidence and commentary, the essay received 4 out of 4 possible points
  • For sophistication of thought, the essay received 0 out of 1 possible points.

This means that the final score for this example essay was a 5 out of 6 possible points . Let's look more closely at the content of the example essay to figure out why it received this score breakdown.

Thesis Development

The thesis statement is one of the three main categories that is taken into consideration when you're awarded points on this portion of the exam. This sample essay received 1 out of 1 total points.

Now, here's why: the thesis statement clearly and concisely conveys a position on the topic presented in the prompt--alternative energy and wind power--and defines the most important factors that power companies should consider when deciding whether to establish a wind farm.

Evidence and Commentary

The second key category taken into consideration when synthesis exams are evaluated is incorporation of evidence and commentary. This sample received 4 out of 4 possible points for this portion of the synthesis essay. At bare minimum, this sample essay meets the requirement mentioned in the prompt that the writer incorporate evidence from at least three of the sources provided.

On top of that, the writer does a good job of connecting the incorporated evidence back to the claim made in the thesis statement through effective commentary. The commentary in this sample essay is effective because it goes beyond just summarizing what the provided sources say. Instead, it explains and analyzes the evidence presented in the selected sources and connects them back to supporting points the writer makes in each body paragraph.

Finally, the writer of the essay also received points for evidence and commentary because the writer developed and supported a consistent line of reasoning throughout the essay . This line of reasoning is summed up in the fourth paragraph in the following sentence: "One thing for power companies to consider is working with turbine manufacturing to make the machines less aesthetically impactful, so as to garner greater public support."

Because the writer did a good job consistently developing their argument and incorporating evidence, they received full marks in this category. So far, so good!

Sophistication of Thought

Now, we know that this essay received a score of 5 out of 6 total points, and the place where the writer lost a point was on the basis of sophistication of thought, for which the writer received 0 out of 1 points. That's because this sample essay makes several generalizations and vague claims where it could have instead made specific claims that support a more balanced argument.

For example, in the following sentence from the 5th paragraph of the sample essay, the writer misses the opportunity to state specific possibilities that power companies should consider for wind energy . Instead, the writer is ambiguous and non-committal, saying, "As with most things, wind power has no easy answer. It is the responsibility of the companies building them to weigh the benefits and consequences."

If the writer of this essay was interested in trying to get that 6th point on the synthesis essay response, they could consider making more specific claims. For instance, they could state the specific benefits and consequences power companies should consider when deciding whether to establish a wind farm. These could include things like environmental impacts, economic impacts, or even population density!

Despite losing one point in the last category, this example synthesis essay is a strong one. It's well-developed, thoughtfully written, and advances an argument on the exam topic using evidence and support throughout.

body-number-four-post-it-note

4 Tips for How to Write a Synthesis Essay

AP Lang is a timed exam, so you have to pick and choose what you want to focus on in the limited time you're given to write the synthesis essay. Keep reading to get our expert advice on what you should focus on during your exam.

Tip 1: Read the Prompt First

It may sound obvious, but when you're pressed for time, it's easy to get flustered. Just remember: when it comes time to write the synthesis essay, read the prompt first !

Why is it so important to read the prompt before you read the sources? Because when you're aware of what kind of question you're trying to answer, you'll be able to read the sources more strategically. The prompt will help give you a sense of what claims, points, facts, or opinions to be looking for as you read the sources.

Reading the sources without having read the prompt first is kind of like trying to drive while wearing a blindfold: you can probably do it, but it's likely not going to end well!

Tip 2: Make Notes While You Read

During the 15-minute reading period at the beginning of the synthesis essay, you'll be reading through the sources as quickly as you can. After all, you're probably anxious to start writing!

While it's definitely important to make good use of your time, it's also important to read closely enough that you understand your sources. Careful reading will allow you to identify parts of the sources that will help you support your thesis statement in your essay, too.

As you read the sources, consider marking helpful passages with a star or check mark in the margins of the exam so you know which parts of the text to quickly re-read as you form your synthesis essay. You might also consider summing up the key points or position of each source in a sentence or a few words when you finish reading each source during the reading period. Doing so will help you know where each source stands on the topic given and help you pick the three (or more!) that will bolster your synthesis argument.

Tip 3: Start With the Thesis Statement

If you don't start your synthesis essay with a strong thesis statement, it's going to be tough to write an effective synthesis essay. As soon as you finish reading and annotating the provided sources, the thing you want to do next is write a strong thesis statement.

According to the CollegeBoard grading guidelines for the AP Lang synthesis essay, a strong thesis statement will respond to the prompt— not restate or rephrase the prompt. A good thesis will take a clear, defensible position on the topic presented in the prompt and the sources.

In other words, to write a solid thesis statement to guide the rest of your synthesis essay, you need to think about your position on the topic at hand and then make a claim about the topic based on your position. This position will either be defending, challenging, or qualifying the claim made in the essay's prompt.

The defensible position that you establish in your thesis statement will guide your argument in the rest of the essay, so it's important to do this first. Once you have a strong thesis statement, you can begin outlining your essay.

Tip 4: Focus on Your Commentary

Writing thoughtful, original commentary that explains your argument and your sources is important. In fact, doing this well will earn you four points (out of a total of six)!

AP Lang provides six to seven sources for you on the exam, and you'll be expected to incorporate quotations, paraphrases, or summaries from at least three of those sources into your synthesis essay and interpret that evidence for the reader.

While incorporating evidence is very important, in order to get the extra point for "sophistication of thought" on the synthesis essay, it's important to spend more time thinking about your commentary on the evidence you choose to incorporate. The commentary is your chance to show original thinking, strong rhetorical skills, and clearly explain how the evidence you've included supports the stance you laid out in your thesis statement.

To earn the 6th possible point on the synthesis essay, make sure your commentary demonstrates a nuanced understanding of the source material, explains this nuanced understanding, and places the evidence incorporated from the sources in conversation with each other. To do this, make sure you're avoiding vague language. Be specific when you can, and always tie your commentary back to your thesis!

body-person-arrows-next

What's Next?

There's a lot more to the AP Language exam than just the synthesis essay. Be sure to check out our expert guide to the entire exam , then learn more about the tricky multiple choice section .

Is the AP Lang exam hard...or is it easy? See how it stacks up to other AP tests on our list of the hardest AP exams .

Did you know there are technically two English AP exams? You can learn more about the second English AP test, the AP Literature exam, in this article . And if you're confused about whether you should take the AP Lang or AP Lit test , we can help you make that decision, too.

Want to improve your SAT score by 160 points or your ACT score by 4 points? We've written a guide for each test about the top 5 strategies you must be using to have a shot at improving your score. Download it for free now:

Get eBook: 5 Tips for 160+ Points

Ashley Sufflé Robinson has a Ph.D. in 19th Century English Literature. As a content writer for PrepScholar, Ashley is passionate about giving college-bound students the in-depth information they need to get into the school of their dreams.

Student and Parent Forum

Our new student and parent forum, at ExpertHub.PrepScholar.com , allow you to interact with your peers and the PrepScholar staff. See how other students and parents are navigating high school, college, and the college admissions process. Ask questions; get answers.

Join the Conversation

Ask a Question Below

Have any questions about this article or other topics? Ask below and we'll reply!

Improve With Our Famous Guides

  • For All Students

The 5 Strategies You Must Be Using to Improve 160+ SAT Points

How to Get a Perfect 1600, by a Perfect Scorer

Series: How to Get 800 on Each SAT Section:

Score 800 on SAT Math

Score 800 on SAT Reading

Score 800 on SAT Writing

Series: How to Get to 600 on Each SAT Section:

Score 600 on SAT Math

Score 600 on SAT Reading

Score 600 on SAT Writing

Free Complete Official SAT Practice Tests

What SAT Target Score Should You Be Aiming For?

15 Strategies to Improve Your SAT Essay

The 5 Strategies You Must Be Using to Improve 4+ ACT Points

How to Get a Perfect 36 ACT, by a Perfect Scorer

Series: How to Get 36 on Each ACT Section:

36 on ACT English

36 on ACT Math

36 on ACT Reading

36 on ACT Science

Series: How to Get to 24 on Each ACT Section:

24 on ACT English

24 on ACT Math

24 on ACT Reading

24 on ACT Science

What ACT target score should you be aiming for?

ACT Vocabulary You Must Know

ACT Writing: 15 Tips to Raise Your Essay Score

How to Get Into Harvard and the Ivy League

How to Get a Perfect 4.0 GPA

How to Write an Amazing College Essay

What Exactly Are Colleges Looking For?

Is the ACT easier than the SAT? A Comprehensive Guide

Should you retake your SAT or ACT?

When should you take the SAT or ACT?

Stay Informed

ap lang rhetorical essay prompts

Get the latest articles and test prep tips!

Looking for Graduate School Test Prep?

Check out our top-rated graduate blogs here:

GRE Online Prep Blog

GMAT Online Prep Blog

TOEFL Online Prep Blog

Holly R. "I am absolutely overjoyed and cannot thank you enough for helping me!”

How to Write the AP Lang Argument Essay (With Example)

December 14, 2023

ap lang argument essay example

We’d like to let you in on a little secret: no one, including us, enjoys writing timed essays. But a little practice goes a long way. If you want to head into your AP English Exam with a cool head, you’ll want to know what you’re getting into ahead of time. We can’t promise the AP Lang Argument Essay will ever feel like an island vacation, but we do have tons of hand tips and tricks (plus a sample essay!) below to help you do your best. This article will cover: 1) What is the AP Lang Argumentative Essay? 2) AP Lang Argument Rubric 3) AP Lang Argument Sample Prompt 4) AP Lang Argument Essay Example 5) AP Lang Argument Essay Example: Answer Breakdown.

What is the AP Lang Argument Essay?

The AP Lang Argument Essay is one of three essays included in the written portion of the AP English Exam. The full AP English Exam is 3 hours and 15 minutes long, with the first 60 minutes dedicated to multiple-choice questions. Once you complete the multiple-choice section, you move on to three equally weighted essays that ask you to synthesize, analyze, and interpret texts and develop well-reasoned arguments. The three essays include:

Synthesis essay: You’ll review various pieces of evidence and then write an essay that synthesizes (aka combines and interprets) the evidence and presents a clear argument. Read our write-up on How to Write the AP Lang Synthesis Essay here.

Argumentative essay: You’ll take a stance on a specific topic and argue your case.

Rhetorical essay: You’ll read a provided passage, then analyze the author’s rhetorical choices and develop an argument that explains why the author made those rhetorical choices. Read our write-up on How to Write the AP Lang Rhetorical Essay here.

AP Lang Argument Essay Rubric

The AP Lang Argument Essay is graded on 3 rubric categories : Thesis, Evidence and Commentary, and Sophistication . How can you make sure you cover all three bases in your essay? We’ll break down each rubric category with dos and don’ts below:

  • Thesis (0-1 point)

When it comes to grading your thesis, AP Exam graders are checking off a box: you either have a clear thesis or you don’t. So, what crucial components of a thesis will get you your check mark?

  • Make sure your thesis argues something . To satisfy your graders, your thesis needs to take a clear stance on the issue at hand.
  • Include your thesis statement in your intro paragraph. The AP Lang Argumentative essay is just that: an essay that makes an argument, so make sure you present your argument right away at the end of your first paragraph.
  • A good test to see if you have a thesis that makes an argument for your AP Lang Argumentative Essay: In your head, add the phrase “I agree/disagree that…” to the beginning of your thesis. If what follows doesn’t logically flow after that phrase (aka if what follows isn’t an agreement or disagreement), it’s likely you’re not making an argument.
  • In your thesis, outline the evidence you’ll cover in your body paragraphs.

AP Lang Argument Essay Rubric (Continued)

  • Avoid a thesis that merely restates the prompt.
  • Avoid a thesis that summarizes the text but does not make an argument.
  • Avoid a thesis that weighs the pros and cons of an issue. Your job in your thesis is to pick a side and stick with it.
  • Evidence and Commentary (0-4 points)

This rubric category is graded on a scale of 0-4 where 4 is the highest grade. Unlike the rhetorical and synthesis essays, the evidence you need to write your AP Lang Argument Essay is not provided to you. Rather, you’ll need to generate your own evidence and comment upon it.

What counts as evidence?

Typically, the AP Lang Argument Essay prompt asks you to reflect on a broad cultural, moral, or social issue that is open to debate. For evidence, you won’t be asked to memorize and cite statistics or facts. Rather, you’ll want to bring in real-world examples of:

  • Historical events
  • Current-day events from the news
  • Personal anecdotes

For this essay, your graders know that you’re not able to do research to find the perfect evidence. What’s most important is that you find evidence that logically supports your argument.

What is commentary?

In this essay, it’s important to do more than just provide examples relevant evidence. After each piece of evidence you include, you’ll need to explain why it’s significant and how it connects to your main argument. The analysis you include after your evidence is commentary .

  • Take a minute to brainstorm evidence that logically supports your argument. If you have to go out of your way to find the connection, it’s better to think of different evidence.
  • Include multiple pieces of evidence. There is no magic number, but do make sure you incorporate more than a couple pieces of evidence that support your argument.
  • Make sure you include more than one example of evidence, too. Let’s say you’re working on an essay that argues that people are always stronger together than apart. You’ve already included an example from history: during the civil rights era, protestors staged group sit-ins as a powerful form of peaceful protest. That’s just one example, and it’s hard to make a credible argument with just one piece of evidence. To fix that issue, think of additional examples from history, current events, or personal experience that are not related to the civil rights era.
  • After you include each piece of evidence, explain why it’s significant and how it connects to your main argument.
  • Don’t summarize or speak generally about the topic. Everything you write must be backed up with specific and relevant evidence and examples.
  • Don’t let quotes speak for themselves. After every piece of evidence you include, make sure to explain and connect the evidence to your overarching argument.

AP Lang Argument Essay (Continued)

  • Sophistication (0-1 point)

According to the College Board , one point can be awarded to AP Lang Argument essays that achieve a high level of sophistication. You can accomplish that in four ways:

  • Crafting a nuanced argument by consistently identifying and exploring complexities or tensions.
  • Articulating the implications or limitations of an argument by situating it within a broader context.
  • Making effective rhetorical choices that consistently strengthen the force and impact of the student’s argument.
  • Employing a style that is consistently vivid and persuasive.

In sum, this means you can earn an additional point for going above and beyond in depth, complexity of thought, or by writing an especially persuasive, clear, and well-structured essay. In order to earn this point, you’ll first need to do a good job with the fundamentals: your thesis, evidence, and commentary. Then, to earn your sophistication point, follow these tips:

  • Outline your essay before you begin to ensure it flows in a clear and cohesive way.
  • Include well-rounded evidence. Don’t rely entirely on personal anecdotes, for example. Incorporate examples from current events or history, as well.
  • Thoroughly explain how each piece of evidence connects to your thesis in order to fully develop your argument.
  • Explore broader implications. If what you’re arguing is true, what does that mean to us today? Who is impacted by this issue? What real-world issues are relevant to this core issue?
  • Briefly explore the other side of the issue. Are the instances where your argument might not be true? Acknowledge the other side, then return to proving your original argument.
  • Steer clear of generalizations (avoid words like “always” and “everyone”).
  • Don’t choose an argument you can’t back up with relevant examples.
  • Avoid complex sentences and fancy vocabulary words unless you use them often. Long, clunky sentences with imprecisely used words are hard to follow.

AP Lang Argument Sample Prompt

The sample prompt below is published online by the College Board and is a real example from the 2021 AP English Exam. The prompt provides background context, essay instructions, and the text you need to analyze.

Suggested time—40 minutes.

Many people spend long hours trying to achieve perfection in their personal or professional lives. Similarly, people often demand perfection from others, creating expectations that may be challenging to live up to. In contrast, some people think perfection is not attainable or desirable.

Write an essay that argues your position on the value of striving for perfection.

In your response you should do the following:

  • Respond to the prompt with a thesis that presents a defensible position.
  • Provide evidence to support your line of reasoning.
  • Explain how the evidence supports your line of reasoning.
  • Use appropriate grammar and punctuation in communicating your argument.

AP Lang Argument Essay Example

As the old phrase says, “Practice makes perfect.” But is perfection something that is actually attainable? Sometimes, pushing for perfection helps us achieve great things, but most often, perfectionism puts too much pressure on us and prevents us from knowing when we have done the best we can. Striving for perfection can only lead us to shortchange ourselves. Instead, we should value learning, growth, and creativity and not worry whether we are first or fifth best.

Students often feel the need to be perfect in their classes, and this can cause students to struggle or stop making an effort in class. In elementary and middle school, for example, I was very nervous about public speaking. When I had to give a speech, my voice would shake, and I would turn very red. My teachers always told me “relax!” and I got Bs on Cs on my speeches. As a result, I put more pressure on myself to do well, spending extra time making my speeches perfect and rehearsing late at night at home. But this pressure only made me more nervous, and I started getting stomach aches before speaking in public.

Once I got to high school, however, I started doing YouTube make-up tutorials with a friend. We made videos just for fun, and laughed when we made mistakes or said something silly. Only then, when I wasn’t striving to be perfect, did I get more comfortable with public speaking.

AP Lang Argumentative Essay Example (Continued)

In the world of art and business and science, perfectionism can also limit what we are able to achieve. Artists, for example, have to take risks and leave room for creativity. If artists strive for perfection, then they won’t be willing to fail at new experiments and their work will be less innovative and interesting. In business and science, many products, like penicillin for example, were discovered by accident. If the scientist who discovered penicillin mold growing on his petri dishes had gotten angry at his mistake and thrown the dishes away, he would never have discovered a medicine that is vital to us today.

Some fields do need to value perfection. We wouldn’t like it, for example, if our surgeon wasn’t striving for perfection during our operation. However, for most of us, perfectionism can limit our potential for learning and growth. Instead of trying to be perfect, we should strive to learn, innovate, and do our personal best.

AP Lang Argument Essay Example: Answer Breakdown

The sample AP Lang Argumentative Essay above has some strengths and some weaknesses. Overall, we would give this essay a 3 or a 4. Let’s break down what’s working and what could be improved:

  • The essay offers a thesis that makes a clear argument that is relevant to the prompt: “Striving for perfection can only lead us to shortchange ourselves. Instead, we should value learning, growth, and creativity and not worry whether we are first or fifth best.”
  • The first body paragraph provides evidence that supports the essay’s thesis. This student’s personal anecdote offers an example of a time when perfectionism led them to shortchange themselves.
  • The second body paragraph provides additional evidence that supports the essay’s thesis. The example describing the discovery of penicillin offers another example of a situation in which perfectionism might have limited scientific progress.
  • The writer offers commentary explaining how her examples of public speaking and penicillin illustrate that we should “value learning, growth, and creativity” over perfectionism.
  • The essay follows one line of reasoning and does not stray into tangents.
  • The essay is organized well with intro, body, and concluding paragraphs. Overall, it is easy to read and is free of grammar errors.

What could be improved:

  • Although the second body paragraph provides one good specific example about the discovery of penicillin, the other examples it offers about art and business are only discussed generally and aren’t backed up with evidence. This paragraph would be stronger if it provided more examples. Or, if this writer couldn’t think of examples, they could have left out mentions of art and business altogether and included alternate evidence instead.
  • This writer would more thoroughly support their argument if they were able to offer one more example of evidence. They could provide another personal anecdote, an example from history, or an example from current events.
  • The writer briefly mentions the other side of the argument in their concluding paragraph: “Some fields do need to value perfection. We wouldn’t like it, for example, if our surgeon wasn’t striving for perfection during our operation.” Since it’s so brief a mention of the other side, it undermines the writer’s overall argument. This writer should either dedicate more time to reflecting on why even surgeons should “value learning, growth, and creativity” over perfectionism, or they should leave these sentences out.

AP Lang Argument Essay Example—More Resources

Looking for more tips to help you master your AP Lang Argumentative Essay? Brush up on 20 Rhetorical Devices High School Students Should Know and read our Tips for Improving Reading Comprehension .

If you’re ready to start studying for another part of the AP English Exam, find more expert tips in our How to Write the AP Lang Synthesis and How to Write the AP Lang Rhetorical Essay blog posts.

  • High School Success

' src=

Christina Wood

Christina Wood holds a BA in Literature & Writing from UC San Diego, an MFA in Creative Writing from Washington University in St. Louis, and is currently a Doctoral Candidate in English at the University of Georgia, where she teaches creative writing and first-year composition courses. Christina has published fiction and nonfiction in numerous publications, including The Paris Review , McSweeney’s , Granta , Virginia Quarterly Review , The Sewanee Review , Mississippi Review , and Puerto del Sol , among others. Her story “The Astronaut” won the 2018 Shirley Jackson Award for short fiction and received a “Distinguished Stories” mention in the 2019 Best American Short Stories anthology.

  • 2-Year Colleges
  • Application Strategies
  • Best Colleges by Major
  • Best Colleges by State
  • Big Picture
  • Career & Personality Assessment
  • College Essay
  • College Search/Knowledge
  • College Success
  • Costs & Financial Aid
  • Dental School Admissions
  • Extracurricular Activities
  • Graduate School Admissions
  • High Schools
  • Law School Admissions
  • Medical School Admissions
  • Navigating the Admissions Process
  • Online Learning
  • Private High School Spotlight
  • Summer Program Spotlight
  • Summer Programs
  • Test Prep Provider Spotlight

College Transitions Sidebar Block Image

“Innovative and invaluable…use this book as your college lifeline.”

— Lynn O'Shaughnessy

Nationally Recognized College Expert

College Planning in Your Inbox

Join our information-packed monthly newsletter.

I am a... Student Parent Counselor Educator Other Zip Code Sign Up Now

Calculate for all schools

Your chance of acceptance, your chancing factors, extracurriculars, ap lang rhetorical analysis essay examples.

Hi everyone! I'm looking for some really good examples of rhetorical analysis essays for AP Lang. Would love to see how others have approached this type of essay and learn from them!

Hello! I'm glad you're looking for examples to learn and improve your rhetorical analysis skills. To guide you, I'll point out a few resources and tips that could be helpful.

1. College Board resources: The College Board, which oversees the AP program, provides sample essays and scoring guidelines on their website. You can find past exam questions and sample responses for AP Lang, including rhetorical analysis essays, in their "Free-Response Questions" section. These examples include the grading commentary that explains what makes the essays effective. Here's the link: https://apcentral.collegeboard.org/courses/ap-english-language-and-composition/exam/past-exam-questions

2. Analyzing other AP Lang prompts: It's also useful to look at the rhetorical analysis prompts from past exams and try to analyze them yourself. Practice identifying the rhetorical strategies used and make notes on what you think works well and what could be improved. This will help you understand the process of writing an effective rhetorical analysis.

3. Check the AP Lang subreddit or online forums: The AP Lang subreddit and other online forums can be a great place to find examples of rhetorical analysis essays written by other students. Be sure to read the comments, as they often contain valuable feedback and insight from peers and experienced individuals. Remember, though, to take these examples with a grain of salt, as they may not be as high quality or well vetted as official College Board examples.

4. Get feedback: As you practice writing your own rhetorical analysis essays, try to get feedback from your teacher, classmates, or an experienced individual who is familiar with AP Lang expectations. This will help you refine your writing and understand what areas you need to improve.

When examining samples, focus on understanding how the writer:

- Clearly states their thesis, identifying the rhetorical strategies they'll analyze

- Organizes their essay, often using a chronological or point-by-point structure

- Analyzes specific examples from the text and explains their significance

- Uses effective transitions between points and examples

- Builds a coherent and well-supported argument throughout the essay

By studying examples and practicing your own writing, you'll be well equipped to tackle the AP Lang rhetorical analysis essay. Good luck!

About CollegeVine’s Expert FAQ

CollegeVine’s Q&A seeks to offer informed perspectives on commonly asked admissions questions. Every answer is refined and validated by our team of admissions experts to ensure it resonates with trusted knowledge in the field.

  • Study Guides
  • Homework Questions

AP+English+Language+and+Composition+-+andquotMother+Tongueandquot+-+Discussion

IMAGES

  1. AP Lang Rhetorical Analysis Essay Example: Prompt- Ghandhi Speech

    ap lang rhetorical essay prompts

  2. 7 Satirical Rhetorical Analysis Prompts, AP English Language, Rhetoric

    ap lang rhetorical essay prompts

  3. AP Lang Rhetorical Essay Student Sample Activity by Jennifer Rodriguez

    ap lang rhetorical essay prompts

  4. AP English Language Rhetorical Analysis Essay Outline by Attis Educates

    ap lang rhetorical essay prompts

  5. ⭐ Ap english language and composition rhetorical analysis essay sample

    ap lang rhetorical essay prompts

  6. AP® Lang Rhetorical Analysis Essay Rubric

    ap lang rhetorical essay prompts

VIDEO

  1. AP Lang

  2. Quick Tip for AP Lang Rhetorical Analysis

  3. AP Lang

  4. How to Plan a Rhetorical Analysis Unit

  5. AP Lang Rhetorical Analysis Rubric

  6. Rhetorical Analysis

COMMENTS

  1. AP English Language and Composition Past Exam Questions

    Download free-response questions from past exams along with scoring guidelines, sample responses from exam takers, and scoring distributions. If you are using assistive technology and need help accessing these PDFs in another format, contact Services for Students with Disabilities at 212-713-8333 or by email at [email protected].

  2. How to Write the AP Lang Rhetorical Analysis Essay (With Example)

    The AP Lang Rhetorical Analysis Essay is one of three essays included in the written portion of the AP English Exam. The full AP English Exam is 3 hours and 15 minutes long, with the first 60 minutes dedicated to multiple-choice questions.

  3. How to Write the AP Lang Rhetorical Essay

    AP Lang Rhetorical Analysis Essay Example Below is a prompt and example for a rhetorical essay, along with its score and what the writer did well and could have improved: The passage below is an excerpt from "On the Want of Money," an essay written by nineteenth-century author William Hazlitt. Read the passage carefully.

  4. AP Lang

    AP Lang Rhetorical Analysis Essay Practice. Rhetorical Analysis practice is one of the most important ways to prepare for the exam! Review student writing practice samples and corresponding feedback from TA Brandon Wu! While you don't need to memorize every rhetorical device for the exam, you should take some time to familiarize yourself with them.

  5. 3 AP® English Language Rhetorical Essay Strategies

    The AP® English Language rhetorical essay can be nightmare inducing for some AP® students, but there is no need for fear. In this exam review we will lay out helpful strategies to get you through the rhetorical essays in no time. Rhetorical Strategy #1: Dissecting the Prompt. The first rhetorical essay strategy is to dissect the prompt.

  6. PDF Sample Student Responses

    AP English Language and Composition Rhetorical Analysis Free-Response Question (2020) Sample Student Responses 1 Sample A [1] To this very day, hate surrounds us. It was no different in the twentieth century, with segregation and rampant wars prevalent. Citizens of all nations were being left behind in rubble and

  7. AP Lang

    Rhetorical Analysis Essay. : A rhetorical analysis essay is an essay that examines how an author uses language and persuasive techniques to convey their message and influence the audience. It analyzes the effectiveness of these strategies in achieving the author's purpose. 🏽 Exam Skills study guides written by former AP English Lang students ...

  8. AP English Language and Composition: Sample Rhetorical Analysis and

    AP English Language and Composition: Sample Argument Question. The following paragraph is adapted from Mirror for Man, a book written by anthropologist Clyde Kluckhorn in the middle of the twentieth century. Read the passage carefully. Then, write an essay that examines the extent to which the author's characterization of the United States ...

  9. AP Lang prompts: How to improve my rhetorical choices?

    7. Practice, practice, practice: Regularly practice writing rhetorical analysis essays. You can find past AP Lang exam prompts online and use these for practice. Ask a teacher or knowledgeable peer to review your work and provide feedback on areas for improvement. 8. Improve your writing skills: As you work on improving your rhetorical analysis ...

  10. Every AP English Language and Composition Practice Exam

    Beginning on page 115, the AP Course and Exam description for AP Language and composition includes 17 multiple-choice questions and three free-response prompts: one synthesis prompt, one analysis prompt, and one argument prompt. As mentioned above, this is the only current source of official practice questions for the new "composition" question ...

  11. SAT® and AP® English Language essay prompts

    Click below to view a list of official SAT® Essay prompts. Then, click the title of each prompt to view the page or download a PDF of the text. 💡Some prompts below include links to an external resource. To include this link as your essay prompt when creating a writing assignment in NoRedInk, be sure to copy and paste the website's full ...

  12. AP Lang Rhetorical Analysis Essay

    The rhetorical analysis essay can seem a bit challenging at first, but with practice and a solid strategy, you can ace it! Here are some tips for tackling this essay: Start by reading the text carefully and taking notes on the author's use of rhetorical strategies, such as ethos, pathos, and logos. Jot down specific examples and make note of ...

  13. How to Write the AP Lang Argument Essay + Examples

    2. Pick one side of the argument, but acknowledge the other side. When you write the essay, it's best if you pick one side of the debate and stick with it for the entire essay. All your evidence should be in support of that one side. However, in your introductory paragraph, as you introduce the debate, be sure to mention any merit the ...

  14. AP® English Prompts and Analysis Resources for Teachers

    The following is a list of our favorite free rhetorical analysis resources for the AP Lang classroom: Stacie Kaminski: A Deep Dive into Rhetorical Analysis. The Garden of English: How to Write a Rhetorical Analysis Essay from Beginning to End. Angie Kratzer: Rhetorical Analysis. Christy's Classroom: Rhetorical Analysis Portfolio.

  15. AP Lang Rhetorical Analysis Essay: Tips & Rubric

    When it comes to AP English language, it involves a section called rhetorical analysis essay. This is a part of three free-response essays that have to be answered within 2 hours and 15 minutes from the overall 3 hours 15 minutes exam. If you are taking the AP Lang exam this year, guidance on how to answer this part will be useful.

  16. How to Write a Perfect Synthesis Essay for the AP Language Exam

    Make rhetorical choices that strengthen your argument and/or employ a vivid and persuasive style throughout your essay. ... The AP Lang exam synthesis essay prompt indicates that you can use generic labels for the sources provided (e.g. "Source 1," "Source 2," "Source 3," etc.). The exam prompt will indicate which label corresponds with which ...

  17. How to Write the AP Lang Argument Essay (With Example)

    Read our write-up on How to Write the AP Lang Rhetorical Essay here. AP Lang Argument Essay Rubric . The AP Lang Argument Essay is graded on 3 rubric categories: Thesis, Evidence and Commentary, ... Typically, the AP Lang Argument Essay prompt asks you to reflect on a broad cultural, moral, or social issue that is open to debate. For evidence ...

  18. AP Lang rhetorical analysis essay examples?

    Hello! I'm glad you're looking for examples to learn and improve your rhetorical analysis skills. To guide you, I'll point out a few resources and tips that could be helpful. 1. College Board resources: The College Board, which oversees the AP program, provides sample essays and scoring guidelines on their website. You can find past exam questions and sample responses for AP Lang, including ...

  19. AP Lang Rhetorical Analysis Essay Example- Prompt: The Perils of

    Thesis: 1 point: The thesis responds to the prompt rather than restating or rephrasing the prompt and clearly articulates a defensible thesis about the rhetorical choices the writer makes by saying Wiesel's ultimate purpose is to persuade world leaders and bystanders to injustices across the world that they should reflect on past mistakes of ...

  20. AP+English+Language+and+Composition+-+andquotMother ...

    AP E NGLISH L ANGUAGE AND C OMPOSITION - "M OTHER T ONGUE" - D ISCUSSION Classification Essay: Mother Tongue - by Amy Tan Organize the information you found by using the SOAPSTone tool to write the claims for a rhetorical analysis of this essay. Study and annotate the text to help you prepare for analysis and writing. Your goal is to determine how Amy Tan uses classification and other ...

  21. Rhetorical Analysis Of George W Bush 9/11

    Rhetorical Analysis Of George W Bush 9/11. Jaelyn An Mrs. Glass AP Language and Composition 15 March 2024 George W. Bush 9/11 Statement Rhetorical Analysis September 11, 2001. A day Americans will never forget. A day this country will never forget. The world witnessed a pivotal moment in history, one that struck the heart of the United States.